#1 Rosh Review

Réussis tes devoirs et examens dès maintenant avec Quizwiz!

Question: Which arteries must be evaluated for aneurysm in any patient with a diagnosed AAA?

Answer: The femoral (85% association) and popliteal (62% association) arteries. Rapid Review Abdominal Aortic Aneurysm (AAA) Advanced age, male, smoking hx, HTN Acute abdominal pain + hypotension + pulsatile abdominal mass US: 100% sensitive CT: 100% sensitive, detects rupture/leak AAA > 5 cm: ↑ risk of rupture Renal colic in elderly: r/o AAA

Question: What feature of tetralogy of Fallot determines the prognosis?

Answer: The prognosis of tetralogy of Fallot depends on the severity of right ventricular outflow tract obstruction. Rapid Review: Tetralogy of Fallot Patient with a history of episodes of cyanosis (tet spells) and squatting for relief PE will show pulmonic stenosis, right ventricular hypertrophy, overriding aorta, VSD CXR will show "bootshaped" heart Comments: Most common cyanotic congenital heart disease Mnemonic: PROVe:: Pulmonic stenosis, Right ventricular hypertrophy, Overriding aorta, VSD

Question: What coronary artery supplies the AV node ventricle?

Answer: The right coronary artery.

Question: What is the greatest risk factor for sudden death from ventricular fibrillation?

Answer: Underlying left ventricular dysfunction (LV EF < 30-35%). Cardiopulmonary Arrest Improved outcomes: AED use Early bystander CPR Presenting rhythm: v-tach/v-fib CPR prior to defibrillation Amiodarone use in shock resistant v-tach/v-fib Therapeutic hypothermia

Question: In a patient with right-sided pelvic pain, what is the imaging modality of choice?

Pelvic ultrasound with Doppler flow. It is sensitive for many GYN disorders and, in the hands of an experienced technician, compressive ultrasound can diagnose appendicitis.

Question: When considering scrotal pain, which signs favor a diganosis of testicular torsion over epididymitis?

Acute severe pain, high-riding testicle, pain worsens with testicle elevation, absent cremasteric reflex, normal testicle but decreased blood flow on color Doppler ultrasonography.

Question: What is the most immediate life threatening complication of pituitary apoplexy?

Adrenal insufficiency. Rapid Review Hypopituitarism Adults: nonfunctioning adenoma most common cause Children: craniopharyngioma most common cause Sheehan syndrome: sudden loss of lactation Apoplexy Hemorrhage into preexisting adenoma HA, AMS, visual dysfunction ​Empty sella syndrome: obese + HTN

Question: What diagnosis must be considered in any patient with persistent, non-healing mouth ulcers?

Answer: Squamous cell carcinoma. Rapid Review Aphthous Ulcers Yellow centers surrounded by red halo Buccal/lip mucosa Recurrent

Question: What is the another name for acute tubular necrosis (ATN)?

Acute Tubular Injury (ATI). Rapid Review Acute Tubular Necrosis Most common cause of intrinsic renal failure Types: ischemic and nephrotoxic Nephrotoxins: aminoglycosides > contrast agents Inability to concentrate urine (urine osmolality = serum osmolality) Granular "muddy brown" casts

Question: What is CREST Syndrome?

Another mixed connective tissue disorder but it is considered more of a constellation of symptoms: Calcinosis, Raynaud's phenomenon, Esophageal dysmotility, Sclerodactyly and Telangiectasias. Rapid Review Scleroderma Excessive collagen deposition F > M Raynaud's phenomenon GI: dysmotility, dysphagia Pulmonary: pulmonary hypertension, interstitial fibrosis Renal: malignant HTN, arteriosclerosis CREST: Calcinosis, Raynaud phenomenon, Esophageal dysmotility, Sclerodactyly, Telangiectasia Systemic: anti-SCL-70 antibody, ANA CREST: anticentromere antibody, ANA

A previously healthy teenage woman presents with a painful mouth ulcer. Examination reveals two discrete ulcers on the buccal mucosa, one 0.5 cm in diameter, and the other 0.8 cm in diameter. She denies a family history of mouth ulcers. Ear, eye, nose and genital examination reveals no abnormalities. Serum laboratory testing is negative for antinuclear antibodies and HLA-B27 antigen, but positive for microcytic anemia. Which of the following is the most likely diagnosis? Aphthous ulcer Behcet's syndrome Reactive arthritis Systemic lupus erythematosus

Answer: (A) Explanation: Although aphthous ulcers have an unclear etiology, they are a common painful condition. There are some risk factors that are associated with aphthous ulcers, such as family history, stress, chemical or physical trauma, food sensitivity and infection. Three types exist: minor, major and herpetiform. Minor qualities include size less than 1 cm, shallow depth, singular or multiple in number and occurring on the labial or buccal mucosa, the floor of the mouth or the soft palate. Major aphthous ulcers are larger, deeper and typically result in scarring. Herpetiform are numerous and vesicular, which may mimic some viral infections. Benign ulcers, no matter which type, are not associated with uveitis, conjunctivitis, arthritis, genital ulcerations, gastrointestinal symptoms, fever or lymphadenopathy. Treatment includes tetracycline, minocycline, over-the-counter benzocaine preparations, viscous lidocaine and topical triamcinolone. Several autoimmune diseases present with oral ulcers that mimic benign aphthous ulcers. Behcet's syndrome, an autoimmune vasculitis, presents with oral and genital ulcers, uveitis and retinitis. Eye examination is normal in this patient, so Behcet's (B) is unlikely. Reactive arthritis (C) (formerly Reiter's syndrome), is a constellation of urethritis, uveitis/conjunctivitis and arthritis, occurs in patients with serum positive HLA-B27. Systemic lupus erythematosus (D) represents an autoimmune attack on connective tissue. There are a multitude of clinical manifestations, including mouth ulcers. Diagnosis is partly dependent on antinuclear antibody (ANA) positivity.

Question: What ankle-brachial index measurement is frequently seen with resting pain?

Answer: < 0.4.

Question: What abnormalities will be seen with hemoglobin and reticulocyte count in aplastic crisis?

Answer: Both hemoglobin and reticulocyte count will be depressed.

Question: What is the name and causative agent of the fifth common childhood exanthemous disease?

Answer: Fifth disease or Erythema Infectiosum caused by parvovirus B19 is the fifth of the classic childhood (slapped cheek)

Question: What are complications of Campylobacter gastroenteritis?

Answer: Guillain-Barre syndrome, reactive arthritis, Reiter syndrome and erythema nodosum. Rapid Review Shigellosis Patient will be complaining of fever, bloody, mucoid diarrhea and seizures (more common in children) Labs will show fecal RBCs and WBCs Treatment is ciprofloxacin Complications: HUS, Reiters syndrome

Question: What risk factors are associated with infection with the mumps virus?

Answer: Lack of immunization, international travel, and immunocompromised status.

Question: Which virus most commonly causes croup?

Answer: Parainfluenza virus. Rapid Review Laryngotracheitis (Croup) Patient will be a non-toxic appearing child, 6 months to 3 years old Complaining of URI symptoms with barky, seal-like cough, inspiratory stridor, low-grade fever Labs will show steeple sign on PA view Most commonly caused by Parainfluenza virus Treatment is steroids, aerosolized epinephrine

Question: What are the most common complications of a molar pregnancy?

Answer: Preeclampsia/eclampsia, pulmonary embolism of trophoblastic cells and hyperemesis gravidarum. Molar Pregnancy: Painless vaginal bleeding Hyperemesis Preeclampsia Large uterus Ultrasound: "snowstorm," "cluster of grapes," "honey-combed" ↑ ß-hcg Dilation and curretage Failure of β-hCG to ↓: choriocarcinoma or invasive disease

Question: List some medications which cause altered level of consciousness?

Anticholinergics, anticonvulsants, antidepressants, antipsychotics, clonidine, illicits (cocaine, heroin, marijuana, ethanol, amphetamines), opioids, salicylates. Dextromethorphan Toxicity: Acts at same receptor as PCP Visual hallucinations Rotary nystagmus May precipitate serotonin syndrome

Question: Antithrombotics include which medications?

Antiplatelets (aspirin, glycoprotein IIb/IIIa, adenosine diphosphate, cyclooxygenase, phosphodiesterase, and thromboxane inhibitors), anticoagulants (vitamin K, factor Xa and thrombin inhibitors) and thrombo-fibrinolytics (plasminogen activators). Acute Coronary Syndrome: Management: Aspirin: ↓ mortality, ↓ infarct size, ↓ reinfarction rate Clopridogrel: patients with aspirin allergy Heparin: ↓ DVT, ↓ reinfarction, ↓ stroke, ↓ LV thrombus, ↓ reocclusion Nitroglycerin: Coronary artery dilation/vascular smooth muscle relaxation → ↓preload/afterload → ↓ myocardial O2 demand Contraindications: sildenafil use within 24 hrs, RV infarction ß-blockers: ↓ Myocardial O2 demand, ↓ ventricular fibrillation IV indications: tachydysrhythmias, intractable HTN Morphine: ↓ Preload/afterload, ↓ sympathetic activity No mortality benefit Glycoprotein IIb/IIIa inhibitors: benefit in patients undergoing PCI PCI: Preferred over thrombolytics in all STEMI patients PCI center: <90 minutes contact to device time Non-PCI center: transfer to PCI center if contact to device time can be <120 minutes Non-PCI center: thrombolytics if contact to device time to be >120 minutes Thrombolytics: begin within 30 minutes of ED arrival if selected

Question: How long is the incubation period for malaria infection?

Approximately 12-14 days.

Question: What dysrhythmia is most commonly associated with tricuspid regurgitation?

Atrial fibrillation (80% of patients with tricuspid regurgitation). Rapid Review Tricuspid Regurgitation: Causes: tricuspid ring stretching > pulmonary HTN, endocarditis, rheumatic heart disease Pansystolic murmur at left sternal border JVP: giant c-v wave Atrial fibrillation

Question: In which clinical scenarios is breastfeeding contraindicated?

Breastfeeding is contraindicated when the mother has HIV or human T-cell lymphotropic virus type I or type II, or the infant has type 1 galactosemia.

Question: Tarsal tunnel syndrome is analogous to which other nerve entrapment pathology?

Carpel tunnel syndrome.

A 70-year-old woman presents to the Emergency Department with the complaint of blisters on her abdomen for the past month. Initially the rash was composed of small pruritic papules which developed into bullae after three weeks. Some of the blisters recently broke and are now tender eroded lesions as shown in the above image. There is no mucosal involvement. When lateral pressure is applied, the bullae do not enlarge. She denies trauma, exposure to new chemicals, starting new medications, or recently hiking outdoors. What is the most likely diagnosis? Bullous pemphigoid Pemphigoid vulgaris Pustular psoriasis Stevens-Johnson syndrome

Correct Answer ( A ) Explanation: Bullous pemphigoid is the most common bullous autoimmune disease of the elderly. The age of onset is between 60-80 years old and it affects men and women equally. The lesions begin as pruritic papules, which early in the disease course can be easily confused with urticaria. However, unlike urticaria, they do not change position. The papules coalesce into large tense bullae over weeks to months and exhibit a negative Nikolsky sign (they do not easily extend when lateral pressure is applied). The bullae appear on normal or erythematous skin and eventually rupture leaving tender eroded lesions. The underlying cause is the production of autoantibodies to basement membrane proteins, basal keratinocyte hemidesmosomal antigens, which cause a detachment at the basement membrane between the epidermis and dermis. Commonly affected body regions include the axilla, medial aspect of the thigh, groin, abdomen, forearm flexors, and lower extremities. They may also be generalized. Mucous membranes are almost never involved. The painful eroded lesions crust and eventually heal spontaneously. Oral steroids are the mainstay of treatment and help to hasten resolution. Recurrence is rare but can occur. Bullous Pemphigoid: Patient will be older than 60 Complaining of intensely pruritic papules that became large, tense blisters/bullae PE will show tense and firm blisters that do not extend with lateral pressure (Nikolsky sign negative) Most commonly caused by chronic autoimmune blistering disease Treatment is corticosteroids and immunosuppressants Pemphigus vulgaris (B) is a generalized mucocutaneous autoimmune blistering eruption characterized by intraepidermal acantholytic blistering due to loss of cohesion between keratinocytes in the epidermis. The vesicles and bullae, which are randomly scattered and arise on normal skin, form with serous content and are Nikolsky sign positive. Commonly affected areas include the head, trunk, and mucous membranes. Pustular psoriasis (C) is characterized by pustules, instead of the scaly plaques of psoriasis vulgaris, that arise on normal or inflamed skin. Generalized acute pustular psoriasis is a rare life-threatening problem with an acute onset and produces burning erythema with pinpoint sterile yellow pustules in clusters that spread throughout the entire body in hours. The pathogenesis of pustular psoriasis is unknown. Stevens-Johnson Syndrome (D) (SJS) is an acute life-threatening mucocutaneous reaction characterized by extensive necrosis and epidermal detachment involving < 10% total body surface area. A hallmark of SJS is that the positive Nikolsky, vesiculobullous lesions are most often found on mucosal surfaces, including the mouth, eyes, vagina, urethra, and anus. Medications most often associated with SJS are antibiotics and anticonvulsants. Two drugs of note are sulfamethoxazole/trimethoprim and lamotrigine due to their classic association with SJS. Unfortunately, the inciting agent is not identified in about half of all cases.

A newborn is being evaluated for cyanosis. Physical examination shows a prominent right ventricular impulse, a systolic thrill, and clubbing of the fingers and toes. A crescendo-decrescendo murmur with a harsh systolic ejection quality is heard along the left upper sternal border. Based on these findings, which of the following is the most likely diagnosis? Tetralogy of Fallot Transposition of the great arteries Tricuspid atresia Truncus arteriosus

Correct Answer ( A ) Explanation: Congenital heart diseases that present with cyanosis include tetralogy of Fallot (TOF), tricuspid atresia, truncus arteriosus, total anomalous pulmonary venous return (TAPVR), and transposition of the great arteries (TGA). Based on the constellation of findings, this newborn most likely has TOF. TOF presents with a ventricular septal defect (VSD), right ventricular hypertrophy (RVH), an overriding aorta, and right ventricular outflow tract obstruction from infundibular stenosis. The murmur in TOF is primarily due to the right ventricular outflow tract obstruction and is usually heard as a crescendo-decrescendo murmur with a harsh systolic ejection quality. The classic pathognomonic finding on chest X-ray is the "boot shaped" heart with an upturned apex. Transposition of the great arteries (B) is an abnormal spatial arrangement of the pulmonary artery and aorta. Cardiac auscultation will typically reveal a loud second heart sound due to aortic valve closure. Tricuspid atresia (C) is complete absence of the tricuspid valve and is typically associated with a holosystolic murmur (due to a VSD). Truncus arteriosus (D) occurs when blood is pumped from the heart through a single truncal valve into a truncal artery, which gives rise to the aorta and the pulmonary arteries. Chest X-ray typically shows an enlarged heart and increased pulmonary vascular markings.

A 15-year-old boy presents with scrotal pain and fever. Examination reveals a tender, swollen testicle. Scrotal Doppler ultrasonography shows increased blood flow to the testicle. Elevation of the scrotum lessens the patient's pain. Which of the following organisms is the most common cause of this condition? Chlamydia trachomatis Escherichia coli Group A Streptococcus Staphylococcus epidermidis

Correct Answer ( A ) Explanation: Epididymitis occurs most commonly in men between the ages of 14 and 35 years. Epididymitis is characterized by a gradual onset of scrotal pain, fever and urinary urgency, frequency, dysuria, pyuria or hematuria. Examination usually reveals localized epididymal edema and tenderness (posterior aspect of scrotum), possible testicular tenderness, and a normal cremasteric reflex. Pain may be relieved with testicular elevation (positive Prehn sign). Scrotal pain should be initially evaluated with a color Doppler ultrasound test, and in the case of epididymitis, the typical findings are an enlarged, thickened epididymis with increased blood flow. The most common organism responsible for epididymitis in those 14 to 35 years-of-age are Neisseria gonorrhoeae and Chlamydia trachomatis. In older individuals (traditionally > 35 years of age), the gram-negative rod bacteria (Escherichia, Klebsiella, Enterobacter and Citrobacter species) are most common. Rapid Review Epididymitis Patient will be complaining of gradual onset unilateral scrotal pain PE will show increased color flow on doppler, relief with testicular elevation (Prehn's sign) Most commonly caused by < 35 yo: C. trachomatis, N. gonorrhea > 35 yo: E. coli, Pseudomonas Treatment is < 35 y/o ceftriaxone/doxycycline, > 35 y/o ciprofloxacin Escherichia coli (B) is more common in men with epididymitis over the age of 35 years. Group A Streptococcus (C) is not associated with deep genitourinary infections. It is however a common cause of superficial epidermal conditions such as cellulitis, impetigo, erysipelas and scarlet fever. Staphylococcus epidermidis (D) is a more common cause of urinary tract infection secondary to external devices, such as a urinary catheter.

A new first-time mother calls for advice on nipple pain with breastfeeding. She is 6 days post partum after an uncomplicated delivery. Which one of the following is most effective for reducing the pain? Education on positioning Expressed breast milk Hydrogel dressing Tea bag compresses

Correct Answer ( A ) Explanation: Nipple pain with breastfeeding is extremely common, with some studies reporting a prevalence of up to 96%. Preventing or alleviating nipple pain is important for comfort, but also for promoting breastfeeding in general. The best intervention for nipple pain is education on proper positioning and attachment of the infant. This position may be lying on her side on the hospital bed or sitting in a comfortable chair. The most common position involves cradling the infant next to the breast from which he or she will feed, with his or her head propped up by the mother's arm. The infant should be placed with his or her stomach flat against the mother's upper abdomen, in the same plane. Topical remedies such as hydrogel dressing (C) and tea bag compresses (D) may also be effective, although no one topical agent has been shown to be clearly superior, and none is as effective as education on positioning and latch-on. Areolar engorgement is treated by the manual expression (B) or pumping of milk to soften the areola and allow better latch-on.

A 35-year-old-man with a history of occasional bloody diarrhea and abdominal pain presents with acute onset severe abdominal pain. Vital signs are significant for a temperature of 39°C, heart rate 140, and blood pressure 82/55 mm Hg. On physical exam, his abdomen is distended and tympanic. Which of the following diagnostic studies is indicated first at this time? Abdominal radiograph Barium enema CT scan of the abdomen pelvis with IV contrast Sigmoidoscopy

Correct Answer ( A ) Explanation: The patient's presentation is suspicious for toxic megacolon, a complication of inflammatory bowel disease. The hallmark of toxic megacolon is colonic dilatation in a patient with a known inflammatory condition of the colon who appears systemically toxic. Patients usually have experienced symptoms of colitis for several days before the onset of toxic megacolon. Diagnosis is made with an abdominal radiograph that reveals dilatation of the colon with a diameter >6 cm. The presence of inflammation and toxicity differentiates toxic megacolon from other disorders that cause colonic dilatation (mechanical obstruction, pseudo-obstruction, and congenital megacolon). Barium enema (B) is contraindicated if toxic megacolon is suspected; it may precipitate an ileus. CT scan of the abdomen and pelvis with IV contrast (C) would not be the first imaging modality of choice; the diagnosis can be easily made on plain film. Sigmoidoscopy (D) is the most sensitive method for establishing the diagnosis of ulcerative colitis, but this is not an appropriate study in the setting of an acute surgical abdomen.

Which of the following correctly describes a positive straight leg raise? Pain in the back which radiates past the knee when raising the symptomatic leg 45 degrees Pain in the back which radiates past the opposite affected knee when raising the asymptomatic leg 45 degrees Pain in the back without radiation to the leg when raising the asymptomatic leg 45 degrees Pain in the back without radiation to the leg when raising the symptomatic leg 45 degrees

Correct Answer ( A ) Explanation: The straight leg test is an easy test to assess for the presence of an L5 or S1 radiculopathy. While supine with the legs both extended, the symptomatic leg is passively raised keeping the knee straight. The presence of pain in the back which radiates past the knee when the leg is elevated 30 to 70 degrees is a positive test. The straight leg test has a sensitivity of 91%, but a specificity of only 26%. Pain in the back which radiates past the opposite affected knee when raising the asymptomatic leg 45 degrees (B) is considered a positive crossed straight leg test. This test has a much higher specificity for diagnosing sciatica (88%), but a low sensitivity. Pain in the back without radiation to the leg when raising the asymptomatic leg 45 degrees (C) does not correctly describe the straight leg raise test. Pain in the back without radiation to the leg when raising the symptomatic leg 45 degrees (D) is considered a negative finding.

Which of the following best describes the rhythm seen in the ECG above? Complete heart block First degree heart block Mobitz I (Wenckebach) Mobitz II

Correct Answer ( A ) Explanation: Third degree, or complete, heart block is characterized by absent conduction of all atrial impulses and complete electrical AV dissociation. The hallmark findings of third-degree AV heart block are regular PP intervals unrelated to regular R-R intervals with P waves that appear to march through the QRS-T complexes. There are two independent pacemakers: one in the S-A node and one either in the AV junction (narrow QRS complexes) or within the Purkinje fibers in the ventricles (wide QRS complexes). First degree heart block (B) is defined by normal AV conduction with a prolonged PR interval (>200 ms). There is a 1:1 relationship between P waves and QRS complexes. In Mobitz I (C), the PR interval progressively lengthens and the R-R interval progressively shortens until a beat is dropped. This cycle repeats itself, producing a pattern of "grouped beating." In Mobitz II (D), the PR interval is constant, but there are nonconducted P waves that lead to dropped beats. This block can deteriorate into third-degree heart block.

A 62-year-old man with a history of ongoing tobacco abuse, hypertension, dyslipidemia and erectile dysfunction complains of progressive aching pain in his right buttock and hip. The pain is worse with walking and is relieved with rest. Physical exam of the lower extremities reveals slightly diminished femoral, popliteal, and dorsalis pedis pulses. Which of the following is the most likely site of this patient's peripheral arterial disease? Aortoiliac artery Common femoral artery Popliteal artery Superficial femoral artery

Correct Answer ( A ) Explanation: This patient most likely has peripheral arterial disease in the right aortoiliac artery. Peripheral arterial disease is physiologically significant atherosclerosis of the aortic bifurcations or arteries of the lower limbs. It is strongly associated with smoking, diabetes mellitus, and aging and shares all the risk factors common to atherosclerosis. This patient presents with right hip and buttock claudication, diminished femoral pulses and erectile dysfunction. This presentation commonly represents atherosclerotic disease within the aortoiliac system and is sometimes referred to as Leriche syndrome. Classic claudication is characterized by leg pain that is consistently reproduced with exercise and relieved with rest. The degree of symptoms of claudication depends upon the severity of stenosis, the collateral circulation, and the vigor of exercise. Patients with claudication can present with buttock, hip, thigh, calf, or foot pain, alone or in combination. The usual relationships are between pain location and corresponding anatomic site of arterial occlusive disease. Peripheral arterial disease in the common femoral artery (B) may cause thigh pain with effort but would not result in erectile dysfunction. Peripheral arterial disease in the popliteal artery (C) would produce pain in the lower one-third of the calf. Peripheral arterial disease within the superficial femoral artery (D) usually produces an effort-related discomfort in the upper two-thirds of the calf. Peripheral vascular disease (PVD)

A 23-year-old man with sickle cell disease presents with chest pain, cough, and fever. His vitals are HR 132, RR 28, BP 110/65, and T 101.1°F. His chest X-ray shows a right lower lobe infiltrate. Hemoglobin is 8 g/dL. Which of the following is the first line management that is most likely indicated? Broad spectrum antibiotics and ICU admission Exchange transfusion Oral antibiotics and follow up with hematology Splenectomy and broad-spectrum antiobitics

Correct Answer ( A ) Explanation: This patient with sickle cell disease (SCD) presents with acute chest syndrome requiring broad-spectrum antibiotics and intensive care unit (ICU) admission. SCD is a genetically determined disease due to an abnormal allele for hemoglobin beta chains. The end result of this abnormality is a sickled cell that is less deformable and can cause increased viscosity and sludging of the blood. Additionally, cells are sequestered in the spleen and the liver leading to destruction. Patients experience chronic hemolysis, vaso-occlusive events, thrombosis and ultimately, end organ injury. Splenic autoinfarction occurs early in life and makes patients more susceptible to encapsulated organisms such as Streptococcus pneumoniae, Haemophilus influenzae and Neisseria meningitides. Acute chest syndrome is the leading cause of death in SCD and is defined as fever, chest pain, and the presence of new pulmonary infiltrates. Management consists of supportive care, supplemental oxygen if needed, broad spectrum antibiotics, and admission to the ICU. Sickle Cell Disease Sickling → vaso-occlusive ischemia ↓ O2, dehydration, acidosis → sickling Newborns: initially asymptomatic (due to ↑ HbF) Most common presentation in infants: dactylitis Aplastic crisis: ↓ Hb + reticulocytopenia, parvovirus B19 Acute chest syndrome Most common cause of death in adults Fever, CP CXR: pulmonary infiltrate Splenic sequestration crisis: rapid splenic sequestration of RBCs → splenomegaly + severe anemia Stroke Aseptic necrosis of the femoral head Dysfunctional spleen → ↑ infection risk Salmonella osteomyelitis S. pneumoniae sepsis: most common cause of death in children Exchange transfusion (B) may be required in the treatment of stroke, refractory priapism and severe acute chest syndrome but is not first line therapy. These patients are too sick to be treated with oral antibiotics (C) and require close monitoring for decompensation. Splenectomy (D) is not necessary in SCD as they have likely already autoinfarcted their spleen by adulthood. Splenectomy is one of the treatments for ß-Thalasemia major.

Which of the following diagnostic criteria differentiate Tourette syndrome from other neurological disorders? Childhood onset Comorbid symptoms of depression Developmental delay Observation of tics on physical exam

Correct Answer ( A ) Explanation: Tourette syndrome (TS) is a neurological disorder characterized by the presence of vocal or motor tics. Vocal tics are involuntary utterances that may range from noises to words or sentences. Motor tics are sudden, brief intermittent movements such as facial grimacing, shoulder shrugging, eye blinking or head jerking. One diagnostic criterion for TS defined by the Diagnostic and Statistical Manual of Mental Disorders (DSM-5) is onset before age 18 years. Other criteria for diagnosis include the presence of multiple motor and one or more vocal tics, persistence of tics for more than one year, and the symptoms not being related to a medical condition or physiologic effects of a substance. TS is found more commonly in boys than girls. Treatment includes patient education, dopamine agonists or antagonists, and botulinum toxin injections. Most tics in patients with TS resolve by age 18 years. Depression (B) and anxiety are commonly seen in the general patient population and are not more common in patients with TS. Developmental delay (C) is not generally seen in patients with TS and is likely related to other disorders such as intellectual disability, cerebral palsy, autism spectrum disorder or Down syndrome. Many patients with TS do not demonstrate tics on the first office visit (D). Aids to diagnosis include interviewing several sources about patient behavior and instructing the patient and parents to videotape symptoms for later evaluation.

Elective surgical repair should be offered to which of the following patients with asymptomatic abdominal aortic aneurysm? 22-year-old man with Ehlers-Danlos syndrome; aneurysm size 3.5 cm 42-year-old man with hypertension; aneurysm size 3.9 cm with 0.9 cm expansion in 6 months 52-year-old man with positive family history; aneurysm size 4.5 cm 62-year-old man with tobacco use; aneurysm size 4.5 cm with aneurysm growth 0.2 cm in two years

Correct Answer ( B ) Explanation: Abdominal aortic aneurysm (AAA) is defined as any infrarenal aortic diameter ≥ 3 cm. It occurs in men 5-10x more than women. Risk factors include family history (12-19% of first-degree relatives are affected), male sex, age, atherosclerosis, tobacco use and hypertension. The underlying pathology involves oxidative stress, aortic wall inflammation and proteolytic degradation of elastin and collagen. Abdominal ultrasound screening is recommended in any male aged 65-75 years who has ever smoked. Most AAAs are asymptomatic and found incidentally during other diagnostic testing. However, if found, AAAs must be monitored, as size, and not comorbidity, usually dictates management. Elective surgical correction of an asymptomatic AAA is offered to patients when an AAA grows ≥ 5.5 cm, or grows more than 0.6 to 0.8 cm over 6 months. Therefore, the 42-year-old man with hypertension aneurysm size 3.9 cm with 0.9 cm expansion in 6 months should undergo elective repair. Those with AAAs < 5.5 cm, or growth ≤ 0.6 cm per year (A, C and D) are followed with serial ultrasound monitoring, no matter what the age or underlying comorbidity. The following are guidelines, and need alteration if risk factors are present: < 3 cm (no further testing), 3-4 cm (every 12 months) and 4-4.5 cm (every 6 months). Refer to a vascular specialist if an AAA is > 4.5 cm. It is, however, important to medically maximize these patients in an attempt to decrease the likelihood of rupture and to properly prepare for possible future surgery. This includes tobacco cessation, blood pressure control, LDL lowering to < 70 mg/dL and oral beta-blockade.

A urine microscopy of a patient with an elevated creatinine reveals muddy brown cellular casts. What is the most likely diagnosis? Acute interstitial nephritis Acute tubular necrosis Nephrolithiasis Pyelonephritis

Correct Answer ( B ) Explanation: Acute tubular necrosis (ATN) is responsible for more than 50% of cases of acute kidney injury in the hospital setting. ATN occurs when there is decreased blood flow to the kidney usually secondary to a drop in blood pressure. The lack of perfusion causes ischemia to the renal tubules, and therefore necrosis. The most significant finding on urine microscopy is muddy brown cellular casts, but additionally, there may be fine granular casts as well as renal tubular epithelial cells. Other urinary labs will reveal a BUN to creatinine ratio of less than 20 as well as a fractional excretion of sodium (FeNa) greater than 2%, both indicative of renal injury. ATN can also be caused by medications, most commonly NSAIDs, ACE inhibitors and angiotensin-receptor blocks (ARBs), which also act to decrease renal blood flow. Acute interstitial nephritis (AIN) (A) is the second most common diagnosis of AKI following ATN. AIN is an acute inflammation of the interstitial space of the kidney most commonly caused by drugs such as antibiotics, antivirals, NSAIDs, PPI's, and diuretics. Urinalysis will reveal hematuria and mild proteinuria. Treatment entails finding and discontinuing the insulting agent, and in some cases, short periods of steroid treatment. Nephrolithiasis (C), or kidney stones, can cause kidney injury by obstructing urine outflow. Patients with this condition are likely to have flank pain, dysuria, and hematuria. Pyelonephritis (D) is an infection of the kidney, which presents with white blood cell casts. This condition can be distinguished from a urinary tract infection by the presence of white cell casts, which are not usually present in an isolated UTI.

What test is diagnostic for primary hypothyroidism? Elevated thyroid stimulating hormone, high free T4, high free T3 Elevated thyroid stimulating hormone, low free T4, low free T3 Low thyroid stimulating hormone, high free T4, high free T3 Low thyroxine binding globulin

Correct Answer ( B ) Explanation: An elevated thyroid stimulating hormone (TSH) level with low free T3 and T4 is diagnostic for primary hypothyroidism. Hypothyroidism is a condition of decreased production or secretion of thyroid hormone from the thyroid gland. It can result from decreased stimulation of the thyroid gland (central or secondary hypothyroidism) or gland dysfunction (primary hypothyroidism). It is a common endocrine disorder and is 4-5 times more common in women than in men. Although there are numerous causes of the disease and varied presentations, an elevated TSH is the most sensitivity and best screening test in the diagnosis of primary hypothyroidism. In primary hypothyroidism, lack of thyroid hormone production leads to decreased negative feedback from the thyroid to the pituitary gland. The pituitary gland, in turns, increases production of TSH in an attempt to increase the thyroid gland's hormone production. A high TSH with a low T4 level is indicative of primary hypothyroidism while the presence of a low TSH and low T4 level is indicative of central hypothyroidism. Elevated thyroid stimulating hormone, high free T4, high free T3 (A) is characteristic of a TSH producing pituitary adenoma. Low thyroid stimulating hormone, high free T4, high free T3 (C) is characteristic of T3 toxicosis which is a rare condition. A low thyroxine binding globulin (D) level can lead to higher circulating T3 levels and hyperthyroidism.

Which of the following anatomical structures is most commonly affected by Rubulavirus? Pancreas Parotid glands Sublingual glands Testes

Correct Answer ( B ) Explanation: Mumps is an acute viral illness caused by the RNA virus, Rubulavirus. Transmission is through respiratory droplets, fomites, or direct contact. It is extremely contagious and can spread rapidly among people sharing close quarters. The characteristic clinical manifestation of mumps is the swelling of salivary glands, most typically the parotid glands. Other symptoms include fever, malaise, headache, and ear pain. Mumps cases have decreased significantly due to widespread vaccination programs, although sporadic outbreaks can occur. Diagnosis is based on the presence of parotiditis along with a history suggestive of mumps. The course of illness is self-limited, with treatment being supportive measures, including ibuprofen or acetaminophen. Prevention of illness is through immunization with the mumps vaccine. Acute pancreatitis (A) is occasionally seen in mumps infection but is not a common manifestation. Bilateral sublingual gland (C) inflammation can occur with mumps but is a less common presentation than parotid gland inflammation. Complications of mumps involving the testes (D) include epididymorchitis, an inflammation of the epididymis and testes.

An elderly man presents with 4 episodes of angina in the past 24 hours. His medical history includes diabetes and advanced COPD. Based on initial testing, you diagnose non-ST-elevation myocardial infarction. You are waiting for the cardiac team to admit him to the critical care unit. In the interim, which of the following is the most appropriate medication to begin? Atelplase Clopidogrel Digoxin Metoprolol

Correct Answer ( B ) Explanation: Non-ST-elevation myocardial infarction (NSTEMI) treatment begins with a basic anti-ischemic regimen consisting of oxygen, morphine, nitrates, and possibly beta-blockers and ACE-inhibitors. Antiplatelet medications are then considered. Choices include aspirin, clopidogrel, and prasugrel. NSTEMI treatment is rounded out with anticoagulants such as enoxaparin, bivalirudin, and fondaparinux. Based on risk stratification, definitive treatment may include medications-alone, angiography, percutaneous cardiac intervention or coronary artery bypass surgery. Clopidogrel acts by irreversibly inhibiting a platelet receptor that is needed for activation, thereby inhibiting platelet function. Thrombolytics (fibrinolytics), such as alteplase (A), reteplase and tenecteplase, are contraindicated in the treatment of NSTEMI, as they have shown worse outcomes with their use. Digoxin (C), a cardiac glycoside, is used in treating certain dysrhythmias and heart failure, not myocardial infarction. Beta-blockers such as metoprolol (D) carry a relative contraindication in patients with severe COPD, asthma, atrioventricular block, hypotension or bradycardia.

Closure of the ductus arteriosus begins when levels of which of the following substances increases in the immediate post-natal period? Adrenocorticotrophic hormone Bradykinin Prostaglandin E1 Pulmonary surfactant

Correct Answer ( B ) Explanation: Patent ductus arteriosus (PDA) is one of the acyanotic congenital cardiac defects. Prematurity is a major risk factor. It is also associated with Down's syndrome and maternal rubella infection. Fetal blood flow occurs normally through the ductus arteriosus from the pulmonary artery to the aorta, thus bypassing the pulmonary vasculature. When this passageway remains patent after birth, it can lead to symptoms of persistent respiratory difficulty and work of breathing (dyspnea), recurrent respiratory infections, tachycardia, cardiomegaly, bounding pulses (widened pulse pressure), machine-like murmur and poor weight gain. After the first breath, pulmonary vascular resistance decreases and stimulates bradykinin release from the infant's lungs. This causes smooth muscle contraction about the ductus arteriosus, beginning the narrowing process until this structure scars close to become the ligamentum arteriosum. ACTH (A) is not responsible for closing the ductus arteriosus. It is an anterior pituitary hormone that stimulates adrenal gland corticosteroid production and secretion. Maternal prostaglandin E1 (C) is responsible for keeping the ductus arteriosus open. Pulmonary surfactant (D) is a lipoprotein that decreases surface tension in the alveoli, thus preventing atelectasis and increasing pulmonary compliance. It is not involved with PDA.

A 73-year-old woman with a past medical history of osteoporosis, degenerative disc disease, and knee pain presents with a chief complaint of worsening dyspnea and recent falls. Upon physical examination you notice an excessive curve of her thoracic spine along with forward-head posture. What is the most likely diagnosis? Spinal hyperlordosis Spinal kyphosis Spinal scoliosis Spinal stenosis

Correct Answer ( B ) Explanation: Spinal kyphosis is the excessive convex curvature of the thoracic spine that tends to progress with age and can lead to complications such as diminished respiratory function and frequent falls, if not corrected. Patients with kyphosis have an excessive rounding of the back that creates a hunchback appearance. Their heads appear to be displaced forward, and often patients can't fully look upward. Patients may complain of back pain and stiffness. Morbidities associated with kyphosis include impaired respiratory function with restrictive respirations, poor physical function, and frequent falls that lead to increased rate of fractures. Diagnosis is made by clinical examination as well as imaging to measure the extent of spinal curvature. Treatment includes symptom management, rehabilitation to retrain the muscles, and surgical realignment. Spinal hyperlordosis (A) is the excessive inward curvature of the lumbar spine. The cervical and lumbar spines have a natural inward curvature, but when this is excessive it is known as swayback, which can lead to lower back pain. The frontal tilt of the pelvis leads to a pelvic tilt, elongating the extensor muscle groups and shortening the flexor muscle groups. Treatment includes strengthening the extensor muscle groups and symptomatic treatment for back pain. Spinal scoliosis (C) is the lateral curvature of the spine that can occur during rapid growth spurts and cause significant spinal deformity. It typically is first noticed in childhood and usually does not cause pain. Scoliosis is diagnosed by physical examination, special tests, and imaging. Treatment involves observation, bracing, and physical therapy to treat muscular imbalances and surgical interventions to realign the spine. Spinal stenosis (D) is the narrowing of the spinal canal caused by degenerative arthritis, movement of a vertebral body, or a lesion. Narrowing of the spinal canal causes impingement on the nerves, which results in pain with activity, weakness, or sensory loss. An MRI is used to make the diagnosis, and treatment includes nonsurgical options such as physical therapy and surgical options such as spinal decompression.

Which of the following is classically seen in flexor tenosynovitis? Extended position of the involved digit Fusiform swelling of the digit Tenderness over the extensor sheath Vesicular eruption over the flexor surface

Correct Answer ( B ) Explanation: The flexor tendons of the fingers are covered by a double layer of synovium to promote gliding of the tendon underneath. Infections in the synovial spaces in the hand tend to spread along the course of the flexor tendon sheaths and may extend proximally to the hand. Infections are usually due to penetrating trauma involving the sheath but occasionally from hematogenous spread. Four cardinal signs of acute flexor tenosynovitis are usually present to help distinguish tenosynovitis from other hand infections. These criteria are referred to as the Kanavel's signs. Flexor tenosynovitis is a surgical emergency. Consultation with a hand surgeon is warranted along with intravenous antibiotics. The affected digit is held in a flexed (A), not extended, posture. The tenderness is over the flexor (C) sheath, not extensor. Vesicles (D) are not commonly associated with flexor tenosynovitis. A localized herpes simplex infection may cause vesicles to form on a digit.

A 15-year-old girl presents to clinic with vaginal discharge. She recently became sexually active but states that her partner does not have any symptoms of infection. Speculum examination reveals white discharge and an erythematous cervix. Bimanual examination is unremarkable. Urine PCR is positive for Chlamydia trachomatis. Which of the following is the most appropriate treatment options? Azithromycin Azithromycin and ceftriaxone Ceftriaxone, amoxicillin and metronidazole Ciprofloxacin and ceftriaxone

Correct Answer ( B ) Explanation: The most common manifestation of chlamydial infection in females is cervicitis and in males, urethritis. Infection may be asymptomatic or may result in vaginal or urethral discharge. The treatment of choice for C.trachomatis cervicitis is a single dose of one gram of azithromycin. In non-pregnant women, seven days of doxycycline is an alternative therapy. Sexual partners should be treated, and each partner should abstain from sexual activity for one week. Follow-up testing is not recommended unless non-compliance is suspected, symptoms persist or return, or the patient is pregnant. Patients diagnosed with cervicitis or urethritis should also be treated for N. gonorrhoeae co-infection with 250 mg ceftriaxone IM. Azithromycin (A) ​alone is inadeuqate due to the high prevelance of chalmydia and gonorrhea co-infection. Amoxicillin (C) does not have a role in the treatment of cervicitis or urethritis. Fluoroquinolones (e.g. ciprofloxacin) (D) have high resistance to N. gonorrhoeae and is no longer recommended.

Which of the following organs is involved in the life cycle of malaria in humans? Brain Heart Kidneys Liver

Correct Answer ( D ) Explanation: The malaria cycle is a complex one involving both the Anopheles mosquito and humans. When an Anopheles mosquito bites a human, sporozoites are transmitted into the human bloodstream. These sporozoites migrate to the liver where they invade hepatocytes, then divide and form multinucleated schizonts. Clinical symptoms are absent during this stage. The schizonts then rupture, releasing merozoites into circulating blood where they invade previously healthy red blood cells. Merozoites mature in human red blood cells, then are ingested by the Anopheles mosquito via mosquito bite. Further maturation from merozoite to sporozoite occurs in the mosquito's midgut, then migration to the mosquito's salivary glands occurs. The cycle is completed by transmission to another host when the mosquito with mature malaria sporozoites bites another human. Encephalopathy in the brain (A) can occur as a result of malaria, but is not part of the malaria life cycle. The heart (B) is not involved in the life cycle of malaria. Renal impairment and failure can occur after malaria infection, but the kidneys (C) are not involved in the maturation process of malaria.

Two weeks after returning from a spelunking trip in Illinois, a previously healthy 25-year-old man presents with a flulike illness. Which of the following is the most likely causative organism? Borrelia burgdorferi Epstein-Barr virus Histoplasma capsulatum Rickettsia rickettsia

Correct Answer ( C ) Explanation: Histoplasma capsulatum is the fungus that causes histoplasmosis. Commonly found in the Ohio and Mississippi river valleys, Histoplasma capsulatum grows in soil enriched with bat or bird droppings. High-risk activities in endemic areas include spelunking, cleaning of chicken coops, excavation, cutting dead trees, and remodeling or demolition of old houses. Inhalation of Histoplasma capsulatum causes infection with a clinical spectrum ranging from asymptomatic to life threatening. Patients present with symptoms similar to the flu including fever, sweats, headache, cough, dyspnea, chest pain, anorexia and myalgia. Treatment recommendations are based on the type of histoplasmosis. Borrelia burgdorferi (A) is a spirochete that causes Lyme disease. The patient history includes exposure to ticks, most commonly in the eastern United States. Physical exam reveals a large, target-shaped skin lesion. Epstein-Barr virus (B) is the causative organism for infectious mononucleosis. Rickettsia rickettsia (D) is transmitted by a tick, flea or mite vector and causes Rocky Mountain spotted fever. All three of these organisms can cause flu-like symptoms similar to histoplasmosis. The patient history and serologic testing help to distinguish the causative organism. Rapid Review Histoplasmosis Patient with a history of travel to Ohio/Mississippi river valleys and exposure to bird/bat droppings X-ray will show solitary pulmonary calcification, hilar and mediastinal adenopathy Diagnosis is made by culture Treatment is itraconazole or amphotericin B

Which of the following is consistent with a diagnosis of infectious mononucleosis? Anterior cervical lymphadenopathy Grayish-white pseudomembrane on the tonsils, uvula, and soft palate Maculopapular rash following treatment with amoxicillin Vesicular lesions and ulcerations on the soft palate

Correct Answer ( C ) Explanation: Infectious mononucleosis is caused by the Epstein-Barr virus. While young children with the disease typically have minimal, if any, symptoms related to the infection, adolescents and young adults usually have a more pronounced course. It is uncommonly seen in adults as most are immune due to a previous exposure. Patients may have a prodrome of low-grade fever, headache, and malaise. There is pharyngeal erythema and tonsillar exudates that may appear white, gray-green, or necrotic. Palatal petechiae can also be seen, but may be present in streptococcal pharyngitis as well. Severe fatigue is common and often the symptom that persists the longest. The lymphadenopathy is usually mildly tender, symmetric and involves the posterior cervical chain. Hepatosplenomegaly can be noted on examination. A generalized maculopapular rash almost always occurs following administration of amoxicillin or ampicillin although the mechanism of this is unclear. Diagnosis is made based on history and physical and can be confirmed with a positive heterophile antibody test (monospot test). Treatment is supportive. Symptoms generally resolve in 1-3 weeks although the fatigue may persist for months. Patients should refrain from contact sports for four weeks post-infection. The lymphadenopathy of mononucleosis typically involves the posterior cervical chain, as opposed to streptococcal pharyngitis which presents with anterior cervical lymphadenopathy (A). Diphtheria can cause a grayish-white pseudomembrane on the tonsils, uvula, and soft palate (B). The exudate seen in mononucleosis does not typically extend to the uvula or palate. Vesicular lesions and ulcerations on the soft palate (D) are a sign of herpes stomatitis.

What is the most common cause of pruritus ani in the pediatric patient? Candida Lichen planus Pinworms Seborrheic dermatitis

Correct Answer ( C ) Explanation: Patients with pruritus ani complain of an uncontrollable urge to scratch the perianal area. In the pediatric population, the pinworm (Enterobius vermicularis) is the most common cause. In adults, the most common case is the presence of feces on the perianal skin. The condition is most often associated with nighttime pruritus. Diagnosis is made by identifying either adult worms migrating in the perineal area or eggs on a clear piece of tape swab of the anus. Treatment for pinworms includes a single dose of mebendazole or pyrantel. Candida (A), lichen planus (B), and seborrheic dermatitis (D) all can cause pruritus ani, but are less common than pinworms in the pediatric population. They are seen more commonly in adults.

A 2-year-old boy presents from a rural Native American reservation with an episode of a 3 minute generalized seizure. On further questioning, he has had 3 days of diarrhea, tenesmus, and fever. He has no prior history of seizures. Which of the following is most likely causing his symptoms? Campylobacter Salmonella Shigella Yersinia

Correct Answer ( C ) Explanation: Shigella species are aerobic, gram-negative bacilli that are primarily transmitted person to person by the fecal-oral route or by ingestion of contaminated food or water. Children in daycare settings or crowded living conditions are most commonly affected. The onset of illness is several days after ingestion. Shigella infects the large intestine and causes watery or loose stools, fever, abdominal cramps or tenderness, tenesmus, and mucoid bloody stools. The characteristic bloody diarrhea appears after the fever subsides. Children under 2 years of age are more likely to develop high fever and seizures. Lab findings are notable for an increased number of bands on the CBC regardless of the actual white blood cell count. Due to high resistance to ampicillin and trimethoprim-sulfamethoxazole, azithromycin in the oral drug of choice for treatment in children. Rare complications include bacteremia, Reiter syndrome, hemolytic-uremic syndrome, toxic megacolon, intestinal perforation and toxic encephalopathy. Salmonella (B) is the most commonly implicated pathogen in food-borne gastroenteritis and usually causes a self-limiting disease but rarely can lead to complications including bacteremia, enterocolitis, meningitis, and osteomyelitis. Campylobacter (A) infections cause diarrhea, abdominal pain mimicking appendicitis or intussusception, malaise and fever but in neonates and young infants bloody diarrhea without fever may be the only manifestation. Yersinia (D) usually manifests as fever and diarrhea in young children and stool often contains leukocytes, blood and mucus. Seizures are not associated with Yersinia.

A 6-week-old infant is brought to the Emergency Room with parental concern for one week of cough and congestion. Although her cough has worsened, she remains afebrile. Chest radiography reveals bilateral interstitial infiltrates and hyperexpansion. What is the treatment of choice? Amoxicillin Cefotaxime Erythromycin Reassurance and return precautions

Correct Answer ( C ) Explanation: The infant has classic findings of pneumonia caused by Chlamydia trachomatis. The onset C.trachomatis pneumonia typically occurs between two and nineteen weeks of life, and almost all affected infants have symptoms by eight weeks of life. Symptoms often begin with nasal congestion and progress to a "staccato cough" and tachypnea. A lack of fever is classic and helps to differentiate chlamydial pneumonia from other causes of pneumonia in this age group. Examination typically reveals rales, but wheezing is uncommon. Chest radiography shows hyperinflation and interstitial infiltrates. The treatment of choice is fourteen days of oral erythromycin. Amoxicillin (A) and cefotaxime (B) are used to empirically treat typical pathogens of community-acquired pneumonia such as S.pneumoniae, M.catarrhalis, S.pyogenes, and H.influenzae. However, viral pneumonia is a more common cause of community-acquired pneumonia than typical bacterial pneumonia during infancy. Moreover, the combination of the patient's age, history (including lack of fever), and radiographic findings make community-acquired pneumonia less likely than C.trachomatis pneumonia. Reassurance and return precautions (D) are appropriate in the setting of both bronchiolitis and viral pneumonia. However, the patient's age, history, and radiography are concerning for C.trachomatis pneumonia.

A 27-year-old man presents with a severe headache and blurred vision. Physical examination reveals bitemporal hemianopsia and an inability to abduct the left eye. What diagnostic modality will most likely yield a diagnosis? CT angiography of the head CT venogram of the head MRI of the brain Noncontrast CT scan of the head

Correct Answer ( C ) Explanation: This patient presents with signs and symptoms concerning for a hemorrhage of the pituitary or pituitary apoplexy and should have an MRI for diagnosis. Pituitary apoplexy describes the presence of a pituitary infarction or hemorrhage leading to decreased pituitary function. Often, patients have a preexisting pituitary tumor. The most common symptoms seen in hemorrhagic pituitary apoplexy are sudden onset of headache caused by subarachnoid bleeding associated with nausea and vomiting. Fever may be present if the vascular supply of the hypothalamus is compromised. Patients typically have ophthalmologic symptoms including decreased visual acuity, opthalmoplegia and visual field defects. The classic visual field defect is bitemporal hemianopsia. Bleeding may cause abnormalities of cranial nerves III, IV and VI. These findings are typically due to leakage of blood into the subarachnoid space or due to mass effect (hemorrhagic or a rapidly expanding mass). Lab testing will reveal global panhypopituitary function. MRI is the optimal study for a number of reasons. The sella turcica, where the pituitary sits, is not imaged well by CT scan due to the amount of bone in the area. Additionally, MRI can differentiate between hemorrhagic and necrotic tissue, which CT cannot. Overall, MRI is 50% more sensitive for apoplexy than CT scan. CT angiography (A) is useful in the detection of arterial aneurysms. CT venogram (B) is helpful in assessing the patency of venous sinuses if cerebral venous thrombosis is suspected. A noncontrast CT of the head (D) is less sensitive than MRI for the previously stated reasons.

A 45-year-old woman presents with low back pain. She states that she has had back pain for years but it is worse today. She reports pain in the low back that radiates to her left foot. Examination reveals no midline tenderness to palpation and a positive straight leg raise. She has no weakness and a normal sensory exam. What diagnostic testing is indicated in the Emergency Department? CT scan of the lumbar spine MRI of the lumbar spine No immediate imaging X-ray of the lumbar spine

Correct Answer ( C ) Explanation: This patient presents with signs of symptoms of sciatica, which may be due to a herniated disk and does not require emergent imaging. Sciatica describes a lumbar radiculopathy that is common seen in individuals. Patients typically complain of sharp, burning or shooting pain beginning in the back and radiating down the leg past the knee. There may be associated numbness or weakness or both on the affected side. The pain is often exacerbated with bending, straining or sitting and relieved with lying supine. Physical examination often reveals tenderness in the sciatic notch and a positive straight leg raise. A straight leg raise is performed with the patient lying supine. The symptomatic leg is then passively raised with the knee fully extended. Eliciting back pain radiating down the leg into the knee at 30 - 70 degrees of leg elevation is suggestive of an L5-S1 radiculopathy. The straight leg raise has a good sensitivity (91%) but poor specificity (26%). Sciatica symptoms are typically caused by lumbar disk herniation (sensitivity of 95%). Weakness of ankle dorsiflexion, toe extension, ankle plantar flexion and knee extension are common findings in disk herniation. Immediate imaging is not indicated in patients with symptoms consistent with disk herniation. The majority of patients with disk herniation and sciatica will improve with conservative management alone. CT scan (A) is an excellent modality for evaluating the presence of fractures of the vertebral column but in the absence of trauma, the yield to this test is low. MRI (B) is the definitive modality in the evaluation of possible cauda equina, spinal infection or malignancy but the patient's presentation is not consistent with these diagnoses. However, the patient in the above clinical scenario does not have any sensory or motor complaints. Plain radiographs (D) have little role in the evaluation of low back pain as they contribute little to the management of these patients.

After return of spontaneous circulation from a ventricular fibrillation arrest, mortality can be determined by calculating a Cardiac Arrest Score, which takes into account which of the following factors? Left ventricular ejection fraction Pre-hospital pharmacologic treatment Systolic blood pressure Time to presentation to the emergency department

Correct Answer ( C ) Explanation: Witnessed out-of-hospital cardiac arrest patients can be stratified by using the 3-criteria Cardiac Arrest Score developed by Thompson and McCullough. The criteria are: (1) Systolic blood pressure in the emergency department; (2) time from loss of consciousness to return of spontaneous circulation; (3) neurologic responsiveness. Criteria are SBP > 90 mm Hg (1 point; < 90 mm Hg = 0 points), time from loss of consciousness to return of spontaneous circulation < 25 minutes (1 point; > 25 minutes = 0 points) and positive neurologic responsiveness (1 point; unresponsiveness = 0 points). A score of 0 equates to 90% in-hospital mortality and 3% chance of neurologic recovery. A score of 3 equates to 18% in-hospital mortality and an 89% chance of neurologic recovery. Left ventricular ejection fraction (A), pre-hospital pharmacologic treatment (B) and time to presentation to the emergency department (D) are not criteria in the Cardiac Arrest Score. However, they do play a role in considering the overall prognosis of cardiac arrest.

Which of the following is used to measure the pulmonary artery wedge pressure? Arterial catheter Flexible bronchoscope Foley catheter Swan-Ganz catheter

Correct Answer ( D ) Explanation: A Swan-Ganz catheter, or pulmonary artery catheter, can be used to obtain the pulmonary artery wedge pressure, which is an estimate of left atrial pressure. The Swan-Ganz catheter permits the clinician to directly measure pressures and sample blood from the right atrium, right ventricle, and pulmonary artery. It can also indirectly measure left atrial pressure. This involves inflating a balloon around the tip of the catheter which occludes a branch of the pulmonary artery, using the artery as an extension of the catheter. Assuming a normal pulmonary artery this technique estimates pressures in the left atrium. Pulmonary artery catheterization can be used for a variety of clinical purposes. It can differentiate among the types of shock, identify etiologies of respiratory and cardiac failure, and diagnose and manage a range of conditions in critically ill patients as well as measure the cardiac output or detect a left-to-right intracardiac shunt. An arterial catheter (A) is a thin, hollow, tube which is placed into an artery, most commonly the wrist or groin, and is indicated when blood pressure must be monitored on a moment-to-moment basis or frequent arterial blood gases are necessary. A flexible bronchoscope (B) is an instrument that is inserted through the mouth or a nasal orifice used to visualize the vocal cords or tracheobronchial tree. It can also be used to sample lesions within the airways. A foley catheter (C), or urinary bladder catheter, is used for urinary drainage or as a means to collect urine for measurement

Which of the following findings seen on rectal examination is most consistent with a concomitant systemic process? Anal fissure with bleeding Anal fissure with deep ulcer Anterior midline anal fissure Lateral anal fissure

Correct Answer ( D ) Explanation: A lateral anal fissure is concerning for an underlying systemic illness in the patient. Most anal fissures occur in the midline. Fissures that occur in other locations should raise concern for diseases like Crohn's, HIV, leukemia, tuberculosis or syphilis. Most commonly, fissures develop from a superficial tear in the anoderm when hard stool is passed often associated with constipation. They occur in areas of weakness of the muscle fibers most commonly in the posterior midline but also in the anterior midline. Patients complain of minor rectal bleeding and significant pain with defecation. An anterior midline anal fissure (C) is the second most common location for a fissure, and occurs more often in women than men. If fissures are not treated, they may develop chronic changes which include a deep ulcer (B), sentinel pile (hypertrophic and edematous skin at the base) and enlarged anal papillae. Bleeding (A) is a common symptom associated with fissures along with significant pain during defecation. Question: What are three treatment options for anal fissures in addition to warm soaks and debulking agents?

You are the primary care provider of an elderly man. Management of his chronic conditions, mainly COPD, has been difficult as he has very limited transportation means and poor access to medical care. You were finally able to get him to see a cardiologist for worsening peripheral edema. After some evaluation, the cardiologist diagnosed chronic cor pulmonale secondary to COPD. She calls you to discuss medication management. In addition to oxygen, which of the following is the most appropriate medication for this patient? Furosemide Iloprost Nifedipine Theophylline

Correct Answer ( D ) Explanation: Acute cor pulmonale requires medical stabilization of the patient, which may include fluid loading, cardiac pressors and vasoconstrictors to maintain proper left ventricular output. For the case of acute cor pulmonale due to massive pulmonary embolism, anticoagulation, thrombolytics and surgical embolectomy are usual therapies. Chronic cor pulmonale is treated with a variety of medical options. The goal is to treat the underlying pulmonary disease and improve oxygenation. Medications are used to improve right ventricular function and decrease pulmonary vasoconstriction. Chronic cor pulmonale regimens consist of supplemental oxygen, diuretics, vasodilators, theophylline, prostacyclin and prostacyclin analogues, endothelin receptor antagonists and phosphodiesterase-type 5 inhibitors. Theophylline is recommended for chronic cor pulmonale due to secondary pulmonary hypertension, as in the case of COPD. It has many benefits, including bronchodilation, inotropism, mild pulmonary vasodilation and enhanced diaphragmatic contractility. It has also been suggested to have anti-inflammatory effects that help to control underlying lung diseases. The use of diuretics in the treatment of chronic cor pulmonale can be difficult. If not used cautiously and with strict monitoring, diuretics, such as furosemide (A), can worsen the disease by causing volume depletion, decreased cardiac output and hypokalemic metabolic alkalosis with decreased ventilatory drive. Diuresis is recommended for the management of chronic cor pulmonale, but it is only to be used with great caution. In patients with poor medical compliance and limited access, medication monitoring can be difficult. This is the case in the above patient, and as such, diuretics are not likely a viable option. Prostacyclin analogues, such as iloprost (B), epoprostenol and treprostinil, are potent vasodilators, and show the best results with chronic cor pulmonale secondary to primary pulmonary hypertension. There is not enough research to recommend the use of this class for cor pulmonale due to secondary pulmonary hypertension due to chronic lung disease such as COPD. Vasodilators, such as calcium channel blockers like nifedipine (C), nitrates, beta agonists and angiotensin-converting enzyme inhibitors, are most beneficial in the management of chronic cor pulmonale secondary to primary pulmonary hypertension. However, they have failed to show benefit in chronic cor pulmonale secondary to COPD, and as such, are not routinely used for these patients.

A 15-year-old boy presents to the ED with an acute onset of altered level of consciousness. There is no history of trauma. He is completely unresponsive to external stimuli, including pain. Which of the following is the most likely cause of coma in this child? Brain malformation Encephalitis Inborn error of metabolism Toxic ingestion

Correct Answer ( D ) Explanation: Consciousness is a state of awareness of one's self and the environment. Coma is a state of no response to any stimuli, including pain. Alterations between these two endpoints deteriorate in a pattern of reduced awareness of self, then environment, then an absence of arousal. Confusion is a loss of clear thinking with impaired decision-making and cognition. Disorientation accompanies or follows confusion, and is characterized as being unaware of time, then place, then memory then self. Delirium follows and is marked by mental and motor excitement, irritability and agitation. Lethargy follows, where the patient is in a state of limited responsiveness, even to moderate stimulation. Stupor is marked by deep sleep or unresponsiveness, however, patients will respond to repeated moderate stimuli. Coma follows. The most common cause of acute onset of altered consciousness in the pediatric population is toxic ingestion. Congenital malformations (A) and inborn errors of metabolism (C) present most commonly in infancy and are chronic. Although encephalitis (B) is common in this age, it almost always presents with fever and is subacute in onset.

A 56-year-old man has pain and tingling in the medial aspect of his ankle and the plantar aspect of his foot. He jogs 3 miles daily and has no history of any injury. The symptoms are aggravated by activity and sometimes keep him awake at night. The only findings on examination are paresthesias when a reflex hammer is used to tap just inferior to the medial malleolus. Which of the following is the most likely diagnosis? Diabetic neuropathy Plantar fasciitis Stress fracture Tarsal tunnel syndrome

Correct Answer ( D ) Explanation: Entrapment of the posterior tibial nerve or its branches as the nerve courses behind the medial malleolus results in a neuritis known as tarsal tunnel syndrome. Causes of compression within the tarsal tunnel include varices of the posterior tibial vein, tenosynovitis of the flexor tendon, structural alteration of the tunnel secondary to trauma, and direct compression of the nerve. Pronation of the foot causes pain and paresthesias in the medial aspect of the ankle and heel. Electromyography and nerve conduction velocity studies may be a useful initial tool in evaluating suspected cases of tarsal tunnel syndrome and in confirming the presence of neuropathy. Medical therapy for tarsal tunnel syndrome may consist of local injection of steroids into the tarsal canal. Physical therapy may be of some value in reducing local soft-tissue edema, thereby easing pressure on the compartment. When conservative therapy fails to alleviate the patient's symptoms, surgical intervention may be warranted. The usual site for a stress fracture (C) is the shaft of the second, third, or fourth metatarsals. Plantar fasciitis (B) is the most common cause of heel pain in runners and often presents with pain at the beginning of the workout. The pain decreases during running only to recur afterward. Diabetic neuropathy (A) is usually bilateral and often produces paresthesias and burning at night, with absent or decreased deep tendon reflexes.

One of your patients has just been diagnosed with a mixed connective tissue disorder, but she does not know which one. However, she remembers hearing the rheumatologist talk about "too much collagen". Which one of the following autoimmune diseases does this patient most likely have? Polymyositis Sjogren's syndrome Systemic lupus erythematosus Systemic sclerosis

Correct Answer ( D ) Explanation: Progressive systemic sclerosis (SS), sometimes referred to as scleroderma, is a chronic progressive connective tissue disease, marked by excessive skin and organ-epithelial collagen production and deposition. In other words, multisystem fibrosis is present and the body becomes "scarred". It is an autoimmune disease of unknown origin, but is associated more with certain factors, such as exposure to silica, organic solvents, aliphatic hydrocarbons, epoxies, pesticides, vibratory tools, certain cosmetics, cocaine and drugs such as bleomycin and carbidopa. It is a rare disease. Signs and symptoms include fatigue, weakness, inflammatory myopathy, myalgias, arthralgias, dependent edema, joint contracture, pruritic thick skin, dysphagia, vomiting, abdominal cramping, diarrhea, dyspnea, nonproductive cough, atypical chest pain and palpitations. The main presenting manifestations are myalgias, arthralgias and Raynaud's phenomenon, a cold-induced, distal digital arteriolar contraction which leads to pallor, ischemia and ulceration. Skin findings predominate on physical exam, and include face, neck, trunk and proximal arm taut thick skin, skin atrophy, areas of hyper- and hypopigmentation, telangiectasias and Raynaud's findings. Lab abnormalities include increased ESR and CRP, thrombocytopenia and microangiopathic hemolytic anemia, hypergammaglobulinemia and increased creatinine phosphokinase. Abnormal renal indices are present if there is kidney involvement. Antinuclear antibodies are positive and anticentromere antibodies are negative in most patients. All of the choices are mixed connective tissue disorders are autoimmune-induced inflammation and not associated with the deposition of collagen. The potential mechanisms for these disease involves several different pathways, however, a basis of immune system dysfunction exists. This can be seen as dysfunction of apoptosis, immune tolerance, T-cells, cellular display of antigens and immune-complexes, to list a few. However, each disorder has a specific end result. In progressive systemic sclerosis, it is collagen overproduction and deposition. In polymyositis (A), myocyte destruction and necrosis are found. In Sjogren's syndrome (B), exocrine gland dysfunction predominates, and leads to the classic sicca symptoms. In systemic lupus erythematosus (C), widespread inflammation results from destruction of double-stranded DNA molecules.

Which one of the following is associated with an erythematous rash and is caused by a bacterial infection? German measles Measles Roseola Scarlet fever

Correct Answer ( D ) Explanation: Scarlet fever is caused by Group A beta-hemolytic Streptococcus (GABS). The pathogenesis is due to erythrogenic toxins (A, B, and C) and occurs most frequently with pharyngitis. Scarlet fever is manifest with a characteristic sandpaper, confluent, erythematous rash, which is usually self-limited. German measles (A) is caused by a RNA Togavirus, the rubella virus. The United States has been free of endemic rubella disease since 2004. Rubella is a relatively mild disease manifest with a homogenous, maculopapular pink to rust-colored rash lasting three days. It is also associated with fever, mild upper respiratory tract symptoms (sore throat, runny nose), malaise, and lymphadenopathy. Rarely, mild meningoencephalitis, arthritis, and thrombocytopenia occur. The major consequence of rubella infection occurring during pregnancy is severe congenital affliction. Measles (B) or rubeola, is caused by an RNA Paramyxovirus. It is the most infectious of the classic exanthems and usual (prior to the advent of vaccines) childhood diseases. The United States was declared free of endemic measles in 2000. However, because of failure of parents to immunize their children, measles outbreaks have returned to the United States. Measles is a potentially fatal disease characterized by a morbilliform, maculopapular rash that lasts about 10 days. It is associated with a prodrome of high fever, brassy cough, coryza, and conjunctivitis, as well as an enanthem (Koplick's spots). Measles can also be complicated by life-threatening pneumonia and encephalitis. Roseola (C) is also known as Exanthem Subitum and is caused by two DNA viruses, Human Herpes Viruses 6 and 7. Roseola is clinically characterized by a sustained high fever, 104°F to 105°F, and irritability without significant focal signs. The fever abruptly resolves after two to five days and a subtle, rose-pink rash appears on the face, trunk, and extremities. Febrile seizures are the most common complication of roseola.

A 32-year-old man presents with acute onset of a hot, swollen left knee. The right knee and other peripheral joints are unaffected. Skin lesions are absent. Cardiopulmonary, genitourinary and neurologic examinations show no gross abnormalities. Laboratory examination reveals a leukocytosis. An X-ray shows normal joint space and absent osteophytes. Which of the following is the most likely diagnosis? Lyme disease Osteoarthritis Rheumatoid arthritis Septic arthritis

Correct Answer ( D ) Explanation: Septic, or pyogenic, arthritis usually occurs after direct inoculation or hematogenous spread into a joint. In adults, there may be an underlying arthritis, whether degenerative or rheumatologic, in the joint. Common causative organisms in adults are Staphylococcus aureus and Neiserria gonorrhoeae. Joint edema, warmth, decreased range and protected use are common symptoms. It is important during any examination of suspected septic arthritis to search for a primary source, such as a penetrating wound, local abscess or skin infection. Laboratory examination includes WBC, CRP and ESR. A joint aspirate (arthrocentesis) should be sent for culture, Gram stain and fluid analysis. Blood cultures are also routinely drawn. Broad-spectrum intravenous antibiotics are started immediately. Surgical decompression and drainage are considered if a patient doesn't respond to antibiotics in 24-48 hours, or if the affected joint is the hip. Common antibiotic choices include oxacillin, cefazolin, ceftriaxone and aminoglycosides. Lyme disease (A) presents with insidious onset, arthritis, rash and cardiac or neurologic manifestations. Osteoarthritis (B) has radiographic abnormalities of joint space narrowing and osteophytosis. Rheumatoid arthritis (C) is associated with an abnormal ESR, symmetric involvement, distal joint predilection and morning stiffness.

A 40-year-old man who plays weekend baseball as a pitcher presents to the office stating that for the last few months his right shoulder feels like it is "going dead." He states that the pain is progressively worsening and now he is experiencing weakness with overhead activity. He also believes that he is throwing the baseball slower than previously. Which of the following is the most likely diagnosis? Acromioclavicular joint injury Adhesive capsulitis Glenohumeral joint dislocation Rotator cuff tear

Correct Answer ( D ) Explanation: The rotator cuff is made up of the Supraspinatus (abduction), Infraspinatus (external rotation), Teres minor (external rotation), and Subscapularis (internal rotation) muscles. Often remembered by the mnemonic SITS. Rotator cuff impingement and tearing usually begin in the supraspinatus tendon as it passes under the acromion. Patients are usually >50 years of age and will often have significant pain with abduction above the head and internal rotation (reaching up the back). Often times, patients will complain of difficulty brushing their hair or have pain at night when rolling onto their shoulder. The condition can also occur in young patients, particularly baseball pitchers who will often hear a "pop" caused by a tear in the rotator cuff. On exam, the Hawkins and Neer tests are positive. If there is weakness on exam and lack of full improvement with rehabilitative exercises and subacromial corticosteroid injection, you should suspect a tear rather than isolated impingement. Tears are diagnosed by MRI and often require surgical repair. Acromioclavicular injury (A) may be traumatic (fall onto the lateral aspect of the shoulder) or atraumatic (arthritis). A fall on the lateral shoulder can result in a stress or tearing of the acromioclavicular (AC) ligaments resulting in a shoulder separation (not the same as dislocation, which involves the glenohumeral joint). Degenerative arthritis of the AC joint is often seen in patients >50 years of age. Adhesive capsulitis (B), also known as a frozen shoulder, is usually the result from prolonged immobility due to another shoulder injury. It leads to loss of both active and passive motion of the shoulder that usually resolves over a period of 6 months to 2 years. Glenohumeral joint dislocation (C) usually occurs as a result of trauma or from extreme abduction and external rotation (serving a volleyball). On exam, there is often a defect seen at the glenohumeral joint.

A 18-year-old G1P0 woman at 16-weeks presents with vaginal bleeding. She had no prenatal care. Vital signs are unremarkable and physical examination only reveals a small amount of blood in the vaginal vault. A transvaginal ultrasound is performed as seen above. Which of the following managements is most likely indicated? Administer methotrexate CT scan of the abdomen and pelvis Intravenous antibiotics Obstetrics consultation

Correct Answer ( D ) Explanation: This patient presents with vaginal bleeding and an ultrasound consistent with a hydatidiform mole or molar pregnancy requiring obstetrics consultation. Molar pregnancy is a spectrum of diseases characterized by abnormal chorionic villi proliferation. A complete hydatidiform mole refers to the situation in which there is no fetal tissue. In an incomplete mole, there is some fetal tissue along with trophoblastic hyperplasia. Patients with molar pregnancy may present with nausea, vomiting, abdominal pain, and vaginal bleeding. Without ultrasound, it is difficult to differentiate these patients from a threatened miscarriage or ectopic pregnancy. Often, the uterine size is larger than the expected for dates in molar pregnancy and the beta-hCG is higher than expected for dates. Diagnosis is based on characteristic findings on ultrasound. Hydropic vessels within the uterus cause a "snowstorm" appearance. Because of the potential for complications and the non-viability of the pregnancy, dilation and curettage is recommended. Once a hydatidiform mole is diagnosed, a chest X-ray should be obtained as trophoblastic tumors metastasize to lung, liver, and brain. Methotrexate (A) is used for the medical management of early ectopic pregnancies. CT scan of the abdomen and pelvis (B) is unnecessary as the ultrasound confirms the diagnosis. Antibiotics (C) are unnecessary in the management of molar pregnancy.

Which of the following murmurs is associated with an increase in right atrial pressure? Aortic regurgitation Mitral regurgitation Mitral stenosis Tricuspid regurgitation

Correct Answer ( D ) Explanation: Valve disorders are often characterized in terms of stenosis (incomplete opening of the valve, thereby increasing the resistance in blood flow) or regurgitation (an incomplete closure of a valve, resulting in a backflow of blood). The tricuspid valve is 1 of 2 atrioventricular (AV) valves and lies between the right atrium and right ventricle. In tricuspid regurgitation, there is incomplete forward flow from the right atrium into the right ventricle and blood flows from the right ventricle into the right atrium during systole. This backflow of blood results in increased right atrial pressure. Tricuspid regurgitation is caused by right ventricular dilation secondary to pulmonary hypertension, rheumatic heart disease, and infective endocarditis. Patients may complain of fatigue and dyspnea on exertion. On exam, there may be a holosystolic murmur best heard at the xiphoid area adjacent to the left sternal border. The aortic valve is one of 2 semilunar valves (the other being the pulmonary valve) and is positioned between the left ventricle and aortic trunk. Aortic regurgitation (A) results in a backflow of blood and an increase in pressure of left ventricle during diastole. The mitral valve is the 2nd AV valve and is situated between the left atria and left ventricle. Mitral regurgitation (B) causes a backflow into the left atria during systole, increasing left atrial pressure. In mitral stenosis (C), the mitral valve cannot fully open during diastole, resulting in an increase in resistance in blood flow through the valve and increasing the left atrial pressure while decreasing the filling of the left ventricle.

A 2-year-old boy presents with a one-day history of inspiratory and expiratory stridor. According to his mother, he has a fever of 38.3°C at home and a cough that sounds like a barking seal. On evaluation, the child is smiling and playful. He is sitting upright, and his voice is hoarse but not muffled. He is not drooling. His heart rate is 138 beats per minute, respiratory rate 28 breaths per minute, temperature 38.5 ºC rectally, and oxygen saturation of 98% on room air. Which of the following is the most likely diagnosis? Bacterial tracheitis Epiglottitis Foreign body aspiration Retropharyngeal abscess Viral croup

Correct Answer ( E ) Explanation: This child has viral croup. It is the most common cause of stridor in children after the neonatal period. Peak incidence is at two years of age. Symptoms are the result of upper airway inflammation. Patients generally have a viral prodrome preceding onset of croup consisting of viral URI symptoms. A barking cough follows this and lasts for up to four days. Patients can have inspiratory and expiratory stridor. Symptoms are worse at night and typically improve with exposure to cold air. The diagnosis is made clinically. If there is concern for a more serious diagnosis, a soft tissue AP and lateral neck film and PA and lateral chest X-ray can be obtained. The subglottic area on the AP neck film will sometimes show the classic "steeple sign" due to symmetric narrowing of the trachea. Treatment is mainly supportive. In mild croup, supplemental oxygen can be given; fluid losses are replaced as needed, and oral dexamethasone is given. In moderate to severe croup, racemic epinephrine is administered as well. Disposition is guided by response to treatment. Children who receive racemic epinephrine aerosol should be observed in the ED for 3-4 hours following its administration to ensure that they do not return to their pretreatment stridorous state once the racemic epinephrine wears off. Epiglottitis (B) is a bacterial infection of the epiglottis commonly caused by S. pneumonia and H. influenza. Its prevalence has decreased significantly due to routine vaccination for the most common causative agents. Onset of the disease is usually rapid. Patients experience inspiratory stridor, drooling, and dysphagia. It is not generally associated with cough. It is historically associated with a "hot potato" voice. Radiographic findings are significant for an enlarged epiglottis commonly referred to as a "thumbprint" sign. Treatment involves IV antibiotics and definitive airway management in a controlled setting involving anesthesia. It is important to keep patients calm because agitation can lead to worsening symptoms and precipitous loss of the airway. Bacterial tracheitis (A) is a bacterial infection of the lining of the trachea. Causative agents include S. aureus, S. pneumonia, and H. influenza, and it is usually the result of secondary bacterial infection following a viral illness. Patients generally appear toxic. They will have inspiratory and expiratory stridor. They will also have a productive cough with thick sputum. Management is similar to epiglottitis and consists of antibiotics and definitive airway management in a controlled setting involving anesthesia. The patient in this clinical scenario did not appear toxic, and the quality of his cough was different. Foreign body aspiration (C) can result in stridor, depending on the resting location of the foreign body in the airway. Common foreign bodies include peanuts, hot dogs, grapes, and small toys. Symptoms generally occur immediately following the aspiration. This patient had symptoms consistent with an infectious etiology, making foreign body aspiration much less likely. Retropharyngeal abscess (D) is a bacterial infection of the retropharyngeal space. It is frequently a polymicrobial infection. Severe cases result in inspiratory stridor, but it was not a common physical finding in this diagnosis. As with epiglottitis and bacterial tracheitis, patients frequently appear toxic. This was not consistent with the patient in this scenario.

Question: What is the Neer test?

Exam of the shoulder where the examiner performs maximal passive forward flexion with internal rotation while stabilizing the patient's scapula with the other hand. Rapid Review Rotator Cuff Impingement and Tear Supraspinatus (abduction) Infraspinatus (external rotation) Teres minor (external rotation) Subscapularis (internal rotation) + Neer and + Hawkins Pain with brushing hair or teeth Pain at night when rolling onto shoulder Baseball pitchers

A 38-year-old woman was admitted to the intensive care unit four days ago after a suicide attempt from an ibuprofen overdose. Today she exhibits a sudden onset of decreased urine volume, altered mental status and generalized edema. Laboratory findings show increased levels of blood urea nitrogen and creatinine. Which of the following is the most likely cause? Acute kidney injury Diabetic ketoacidosis Heart failure Urinary tract infection

Explanation: Acute kidney injury (AKI), previously referred to as acute renal failure, is an abrupt decline in renal filtration function that is generally reversible. AKI has many different possible causes and is categorized by pre-renal, intrinsic and post-renal etiologies. Pre-renal causes are the most common and can lead to intrinsic injury if not addressed promptly. In hospitalized patients AKI is most commonly seen when patients develop acute tubular necrosis due to sepsis, ischemia or nephrotoxic exposure, such as with a drug overdose. Diagnostic testing for AKI includes complete blood count, kidney function studies, urinalysis and renal ultrasound. Increased levels of blood urea nitrogen and creatinine are the classic laboratory findings in AKI. Initial management is to correct the life-threatening electrolyte and fluid abnormalities along with determination and correction of the original cause of the injury. AKI is life threatening and patients present as being toxic. Patients with diabetic ketoacidosis (B), heart failure (C) and urinary tract infection (D) may have similar symptoms to those with AKI and these conditions may lead to AKI if not treated promptly.

A 40-year-old obese, non-pregnant woman presents with left-sided pelvic pain that has been increasing for two days. She has left adnexal tenderness on pelvic exam. Which of the following would point to a gastrointestinal rather than a gynecologic cause of her pain? A history of constipation A history of nausea and vomiting A history of ovarian cysts A wet mount with sheets of white blood cells

Explanation: Constipation is a risk factor for diverticulitis, inflammation, and microperforation of the wall of a diverticulum. Diverticula are thought to be caused by high colonic intraluminal pressure resulting from low-fiber, high-fat diets. The most common site for diverticulitis is the left lower quadrant. Nausea and vomiting (B) are nonspecific symptoms present in both GI and GYN diseases. A history of ovarian cysts (C) places this woman at higher risk of recurrent cysts and of ovarian torsion. Sheets of white blood cells (D) on a wet mount preparation indicate cervicitis if the patient has no pelvic pain, and pelvic inflammatory disease or possibly tubo-ovarian abscess if she does have pain. Diverticulitis Patient will be complaining of abdominal pain that is localized to the left lower quadrant, fever, nausea, vomiting, and a change in bowel habits PE will show localized guarding, rigidity, and rebound tenderness Diagnosis is made by CT scan Treatment is abx

A 32-year-old woman presents to the ED complaining of right calf swelling for one week. She is one week postpartum following an uneventful full-term vaginal delivery. She denies any chest pain, dyspnea, or cough. On examination, her right calf is 16 cm in circumference; her left calf is 10 cm. Which of the following is the most appropriate test to confirm the diagnosis? D-dimer assay Duplex ultrasound Impedance plethysmography Venography

Explanation: Duplex ultrasonography is currently the first-line imaging study to diagnose deep vein thrombosis (DVT) because of its relative ease of use, the absence of irradiation or contrast material, and a high degree of accuracy. In patients with clinically suspected disease, compression ultrasonography is 95%-99% sensitive for proximal venous thrombus compared with contrast venography. D-dimer (A) levels remain elevated in DVT for about seven days. Patients presenting late in the course, after clot organization and adherence have occurred, may have low levels of D-dimer. Similarly, patients with isolated calf vein DVT may have a small clot burden and low levels of D-dimer below the analytic cut-off value of the assay. This accounts for the reduced sensitivity of the D-dimer assay in the setting of confirmed DVT. In some countries, impedance plethysmography (IPG) (C) has been the initial noninvasive diagnostic test of choice and has been shown to be sensitive and specific for proximal vein thrombosis. However, IPG has several limitations, including insensitivity for calf vein thrombosis, non-occluding proximal vein thrombus, and iliofemoral vein thrombosis above the inguinal ligament. Until the 1980s, venography (D) was the standard examination for DVT. This procedure is now rarely performed because of patient discomfort from needle puncture, the potential for infiltration of contrast agent at the injection site or allergy to the agent, and the cost of time and infrastructure necessary to perform the examination

To achieve a normal penile erection, several neurochemical processes are required. Men who suffer from erectile dysfunction are commonly treated with medications from which of the following drug classes? Angiotensin-converting-enzyme-inhibitors Dihydropholate reductase inhibitors HMG-CoA reductase inhibitors Phosphodiesterase 5 inhibitors

Explanation: Erectile dysfunction (ED) refers to the regular and repeated inability to obtain or maintain an erection. It has several causes, both physical and psychological. Some of these causes are aging, tobacco use, neuropathy and vascular disease (multiple sclerosis, diabetes mellitus), renal failure, cavernosal disorders, prior surgery and selective serotonin reuptake inhibitors. During a normal erection, neurologic input to the penis causes a local secretion of nitric oxide, which allows the vessels in the corpora cavernosa to vasodilate, which in turn, results in an influx of blood. A key second messenger in this pathway is cyclic guanosine monophosphate (cGMP). To keep the pathway in check, an enzyme, phosphodiesterase type 5 (PDE5), breaks down cyclic-GMP, reverting the vasodilatory response. In other words, nitric oxide induced, cGMP mediated, penile vasodilation (erection) is inhibited by PDE5. Using the common pharmacologic mantra of "inhibit the inhibitors", the main class of medications used to treat ED works by inhibiting the inhibitor, phosphodiesterase type 5. Angiotensin-converting-enzyme-inhibitors (ACEIs) (A), such as lisinopril, are used to treat hypertension, not ED. Dihydrofolate reductase inhibitors (B), such as methotrexate, are used to treat cancer and autoimmune disease, not ED. HMG-CoA reductase inhibitors ("statins") (C), such as lovastatin, are used to treat hypercholesterolemia. They are not directly used in the treatment of ED. However, angiopathy, a prime cause of ED, is accelerated in patients with hypercholesterolemia. Therefore, some would argue that these medications may be thought of as indirect treatment of ED.

Which of the following is the most common electrocardiogram finding in an acute pulmonary embolism? Right bundle branch block S wave in lead I, Q wave in III, inverted T wave in III (S1Q3T3) Sinus tachycardia T wave inversions in leads V1-V4

Explanation: Sinus tachycardia is the most common ECG abnormality seen in a pulmonary embolism. It is seen in almost half of all cases. A pulmonary embolism most commonly occurs when a deep venous thrombosis (DVT) embolizes from the lower extremities and travels to the pulmonary vasculature. This leads to a ventilation-perfusion mismatch in the lungs resulting in hypoxia. Patients with a pulmonary embolism present with dyspnea, pleuritic chest pain, and tachycardia. Risk factors include malignancy, immobilization, recent surgery, and genetic hypercoagulable states. In cases with a large clot burden, acute pulmonary hypertension occurs and a right heart strain pattern will be seen on ECG. A massive pulmonary embolism is defined as a large clot burden causing obstructive shock. Treatment includes systemic thrombolytics, catheter-directed thrombolytics, and rarely embolectomy. A new right bundle branch block (A) is seen in 18% of pulmonary emboli. It is associated with an increased mortality and a significant clot burden. S1Q3T3 (B) is the 'classic' ECG finding frequently taught although it has a poor sensitivity and specificity. It is found in less than a quarter of cases. T-wave inversions in leads V1-V4 (D) are associated with right ventricular strain. This occurs in a pulmonary embolism as the pressure in the right side of the heart rapidly rises. This causes a dilation of the right side of the heart and a subsequent repolarization abnormality seen as T-wave inversions. Around 18% of patients with PE will have a completely normal ECG.

A 10-year-old boy is brought to the emergency room because of a headache. For the past three days, he has had a fever and a headache. Then, this morning his mother noted the presence of a rash. He denies vomiting, abdominal pain, neck pain, or coughing. Last week, he went hiking with his father although he denies any tick bite. On physical examination, temperature is 38.7℃, heart rate is 88 per minute, and respiratory rate is 23 per minute. There is a blanching erythematous rash with macules on the ankles and wrists. Which of the following is the most likely diagnosis? Leptospirosis Meningococcal meningitis Rocky Mountain spotted fever Thrombotic thrombocytopenic purport

Explanation: The boy has signs and symptoms that are suspicious for Rocky Mountain spotted fever (RMSF). RMSF is usually transmitted via tick bite. The principal vector of RMSF in the eastern and south central United States is Dermacentor variabilis. On the other hand, Dermacentor andersoni is the primary vector in the mountain states west of the Mississippi River. Infected patients become symptomatic two to 14 days after being bitten by an infected tick. Classic symptoms of RMSF include fever, headache, and rash in a person with a history of a tick bite. Rash occurs in approximately 88 percent to 90 percent of patients. The hallmark of RMSF is a blanching erythematous rash with macules that become petechial over time. The appearance of the rash usually begins on the ankles and wrists and spreads to the trunk. The rash that appears on the palms and soles is highly characteristic of RMSF, but usually occurs in later-stage disease. A presumptive diagnosis of RMSF is initially made based upon consistent clinical signs and symptoms in the appropriate epidemiologic setting. The clinical diagnosis must be confirmed through serologic testing or through the use of special stains on a skin biopsy. Leptospirosis (A) generally presents with the abrupt onset of fever, rigors, myalgias, and headache, and conjunctival suffusion. The presentation of meningococcal meningitis (B) consists of the sudden onset of fever, nausea, vomiting, headache, decreased ability to concentrate, and myalgias in an otherwise healthy patient. The petechial rash appears as discrete lesions, most frequently on the trunk and lower portions of the body. Petechiae can coalesce into larger purpuric and ecchymotic lesions. Thrombotic thrombocytopenic purpura (D) usually presents with fatigue, dyspnea, petechiae, or other signs of bleeding.

A 2 1/2-year-old boy presents with a severe sore throat, muffled voice and difficulty opening and closing his mouth. On examination, the oropharynx is markedly erythematous without exudates. A soft tissue X-ray of the neck is obtained and shown. What diagnosis is suggested by the film? Epiglottitis Peritonsillar abscess Retropharyngeal abscess Tracheitis

Explanation: The retropharyngeal space is a potential spaces between the prevertebral fascia and the posterior pharyngeal wall. The lymphatic drainage tissue for the nose, pharynx, sinuses and ears is located within this space. Abscesses may occur in this area from direct trauma, hematogenous seeding or spread from a contiguous infection. Half of pediatric cases occur before the age of one and almost all occur under the age of 6. The diagnosis is difficult to make because of its clinical variability. Patients commonly have fever, sore throat, neck stiffness, trismus, swelling of the neck, drooling and a muffled voice. Depending on the degree of upper airway obstruction, stridor may be present. On soft tissue films of the neck, the retropharyngeal space is enlarged. In general, the width of the retropharyngeal space is roughly equivalent to the width of the vertebra at the same level. Although these are abscesses, not all require surgical intervention and may be managed with intravenous antibiotics. Rapid review with picture... Retropharyngeal Abscess 3-5 years of age S. aureus, GAS, anaerobes, foreign body Sore throat, dysphagia Toxic appearing, drooling, ↓ neck extension, fever Cri du canard (duck "quack") Neck x-ray: widened retropharyngeal space twice the size of the vertebral body CT Rx: IV ABX, ENT consultation Epiglottitis (A) in children is a severe, life-threatening infection. It has become much less common since the development of the H. influenzae type b vaccine. Patients develop fever, severe sore throat and appear toxic. In order to maximize their ability to move air through the upper airway, children may assume a "sniffing" or "tripod" position. On X-ray the enlarged epiglottis may appear like a thumbprint. Manipulation of the airway should be avoided. A peritonsillar abscess (B) occurs more commonly in older children and adult patients. On physical examination, significant tonsillar enlargement with deviation of the uvula away from the site of the abscess is seen. Patients typically require needle aspiration or incision and drainage. Tracheitis (D) is a rare bacterial infection of the trachea causing a constellation of symptoms similar to croup but overlap with the more severe epiglottitis. The tracheal epithelium becomes severely inflamed with the production of thick purulent secretions. Patients require intravenous antibiotics and frequently intubation.

A 65-year-old man presents to the emergency department with chest pain and ST-segment elevation in leads II, III, and aVF. The patient is hypotensive and physical exam reveals jugular venous distention, clear lung fields and tachycardia. No murmur or S3 is appreciated. What is the next step in management? Administer a beta blocker Administer morphine sulfate Administer sublingual nitroglycerin Begin intravenous hydration

Explanation: This patient has an ECG concerning for a right ventricular infarct. In this situation, the patient becomes preload dependent (essentially the right ventricle is impaired so there is "passive" flow into the left ventricle). When these patients become hypotensive, the immediate treatment is intravenous hydration to increase preload. Right ventricular infarction is represented on an ECG by noting ST elevation in right-sided leads (V3R and V4R). Because the right coronary artery often supplies the right side of the heart and the inferior wall of the heart, any ECG with ST elevations in leads II, III, and aVF (inferior leads) should have another ECG performed with right-sided lead placement. Aggressive volume loading with normal saline boluses is used to restore blood flow to the right ventricle and inotropic support is indicated if hypotension persists. Early percutaneous coronary intervention or thrombolytic therapy should be initiated as soon as possible. In general, patients' with right ventricular myocardial infarction are treated in a manner similar to those with acute ST-elevation myocardial infarction. This includes the early use of dual antiplatelet therapy, statin therapy, and an anticoagulant. Medications that lower preload such as nitroglycerin (C) and morphine (B) or drugs that slow heart rate such as beta blockers (A) should be avoided or used with extreme caution.

A 25-year-old pregnant woman is being evaluated for vaginal irritation. Physical exam reveals small, flesh-colored, papillary growths on her vulva as shown. Examination of the cervix is unremarkable. Which of the following is the best initial treatment? Imiquimod cream Podophyllin solution Podophyllotoxin Trichloroacetic acid

Explanation: Trichloroacetic acid is the first-line therapy for genital warts during pregnancy. It can be used for both external and internal warts. Condyloma acuminata, or genital warts, are caused by an infection with the human papillomavirus (HPV) types 6 and 11. It is a sexually transmitted infection that can be prevented with vaccination. Symptoms include flesh colored to white, exophytic or papillomatous growths in the genital area. In women, this can affect the vagina, cervix, vulva, oropharynx, perineum, and perianal areas. HPV can be spread via sexual contact or from mother to newborn during passage through the birth canal. Treatment in pregnancy should be initiated at 32 weeks gestation to allow healing before delivery. Treatment includes trichloroacetic acid solution, podophyllin, cryosurgery, surgical excision, or imiquimod cream application. Imiquimod cream (A), podophyllin solution (B), and podophyllotoxin (C) are all unsafe during pregnancy. Podophyllin is teratogenic.

Question: Greater than what total body surface area (TBSA) is diagnostic of toxic epidermal necrolysis?

Greater than 30%.

Question: How is the patient's voice with a peritonsillar abscess described?

Hot potato.

Question: Which electrolyte imbalance is commonly seen in patients with acute kidney injury?

Hyperkalemia

Question: What are the disease processes that can precipitate toxic megacolon?

IBD (most common), pseudomembranous colitis, CMV colitis, and bacterial colitis. Rapid Review Ulcerative Colitis Chronic inflammatory disease Bloody diarrhea, crampy abdominal pain Continuous mucosal inflammation Always involves the rectum Crypt abscesses Toxic megacolon Colon cancer Sulfasalazine or 5-ASA derivative

Question: Which medication is used to close a patent ductus arteriosus, especially in a premature infant?

Indomethacin, a prostaglandin inhibitor. Rapid Review Patent Ductus Arteriosus (PDA) Fetal period: shunt is right → left (normal) Neonatal period: ↓ lung resistance → shunt reversal (left → right) Auscultation: "machine-like" murmur Close: O2, bradykinin, NSAIDs Open: PGE1

Question: What is the most common vector-borne infection in the United States and Europe?

Lyme disease.

Question: What effect will the administration of atropine have on third degree heart block?

It alters conduction ratios without changing the appearance of the ventricular escape rhythm. Rapid Review Heart Block: Third-Degree Atria and ventricles beat independently P waves " march through" QRS complexes Constant PP intervals Constant RR intervals Avoid atropine Pacemaker

Question: What are three treatment options for anal fissures in addition to warm soaks and debulking agents?

Lidocaine ointment for topical anesthesia, nifedipine gel for reduction of anal canal pressure, and injection of botulinum toxin for relaxation of the anal sphincter. Rapid Review Anal Fissure Patient will be complaining of rectal pain and bleeding which occurs with or shortly after defecation PE will show fissure located in the posterior midline Diagnosis is made by visual inspection Treatment is stool softeners, protective ointments, sitz baths Comments: If fissures are located laterally, search for pathologic etiologies

Question: What is myxedema coma?

Myxedema coma is a the most severe manifestation of hypothyroidism and is defined by altered mental status and decompensated metabolic state. Rapid Review Hypothyroidism 1° > 2° 1° previous radioactive iodine/thyroidectomy > autoimmune 2° most common cause: pituitary tumor Fatigue, weight gain, cold intolerance TSH: ↑ with 1°, ↓ with 2°

Question: What is the most common side effect of single-dose azithromycin?

Nausea

Question: What are complications of flexor tenosynovitis?

Necrosis and proximal spread. Rapid Review Flexor Tenosynovitis Penetrating trauma → S. aureus infection Kanavel's cardinal signs ​Fusiform or symmetrical swelling of finger Flexed posture of finger Tenderness along flexor tendon Pain with passive range of motion ​ABX, surgical emergency

Question: Does neonatal ophthalmic erythromycin prevent infection with Chlamydia trachomatis?

No. Rapid Review Chlamydia Pneumoniae Patient will be a between two and nineteen weeks of life With a history of congestion Complaining of cough, hoarseness and malaise PE will show staccato cough and a lack of fever Most commonly caused by Chlamydia pneumoniae Treatment is macrolide antibiotic

Question: In general, supplemental oxygen is recommended in patients with COPD under which conditions?

PaO2 <55 mm HG or oxygen saturation <88% on room air. Rapid Review Cor Pulmonale Pulmonary HTN + RVH → right heart failure MC chronic cause: COPD MC acute cause: PE Right heart catheterization

Question: What is a reverse straight leg test?

Pain with passive extension of the hip while the patient is prone. It is indicative of an L3 or L4 radiculopathy.

Question: What two conditions promote growth of condyloma acuminatum?

Pregnancy and immunosuppression. Condyloma Acuminata Patient will be complaining of genital lesions PE will show cauliflower-like lesion Most commonly caused by HPV 6 & 11 Comments: most common STD

Question: Name some popular phosphodiesterase type 5 inhibitors?

Sildenafil, vardenafil and tadalafil. PDE-5 Inhibitors Mechanism: inhibits PDE-5 → corpus cavernosum smooth muscle relaxation + ↑ blood flow → erection Headache, dyspepsia ("Hot and heavy, but then headache, heartburn, hypotension") Nitrates + PDE-5 inhibitor use → refractory hypotension

Question: What season(s) does Rocky Mountain spotted fever usually occur?

Spring and early summer.

Question: Neonatal septic arthritis is commonly caused by which organisms?

Staphylococcus aureus and group-B Streptococcus. Haemophilus influenzae should be considered in childhood septic arthritis. Rapid Review Septic Arthritis Age <35: N. gonorrhea S. aureus most common overall Hematogenous spread Fever, pain, ↓ ROM Knee (most common) Arthrocentesis (WBC >50,000 with >75% PMNs) IV ABX, surgical washout

Question: Is there a risk to the fetus of a woman who is infected with mononucleosis?

There is little evidence of teratogenic risk and transplacental transmission appears rare. Rapid Review Infectious Mononucleosis Patient will be complaining of low-grade fever, headache, malaise, severe fatigue PE will show mildly tender lymphadenopathy involving the posterior cervical chain, hepatosplenomegaly Diagnosis is made by heterophile antibody test (monospot test), generalized maculopapular rash following administration of amoxicillin Most commonly caused by Epstein-Barr virus Treatment is self-limiting, refrain from contact sports for four weeks post-infection References:

Question: What is the most common symptom (aside from back pain) seen in cauda equina syndrome?

Urinary retention. Back Pain Night pain, weight loss: malignancy Back pain + fever + neurological deficits: epidural abscess Acute bony tenderness: fracture Young, morning stiffness: seronegative spondyloarthropathy Urinary retention: cauda equina syndrome Pain with extension, relief with flexion: spinal stenosis Image if red flags present

Question: What is the leading cause of secondary healthcare-associated bacteremia?

Urinary tract infection associated with urinary catheters.

Question: What diagnostic test has the highest sensitivity for diagnosing a DVT?

Venography. Deep Vein Thrombosis Unilateral leg swelling Phlegmasia cerulean dolens (painful blue leg) = massive iliofemoral thrombosis with extensive vascular congestion and venous ischemia Phlegmasia alba dolens (painful white leg) = massive iliofemoral thrombosis → arterial spasm Risk stratification: Well's criteria Modality of choice: ultrasound Treatment: ​Proximal DVT: heparin, warfarin Massive DVT: thrombectomy Isolated calf vein thrombosis: aspirin, ultrasound in 2-5 days Recurrent DVT on warfarin: heparin, IVC filter Propagation of DVT on warfarin + heparin: IVC filter

Question: How does coprolalia clinically manifest in patients with Tourette syndrome?

Vocal tic with involuntary use of obscene words. Rapid Review Tourette syndrome (TS) Patient will be a child Complaining of motor or vocal tics PE will show facial grimacing, shoulder shrugging, eye blinking or head jerking Diagnosis is made by clinical criteria Treatment is antidopaminergic medications such as pimozide Comments: Associated with OCD, ADHD

Question: Is spinal kyphosis more common in men or women?

Women

Question: Do all family members need to be treated if one person is diagnosed with pinworms?

Yes. Rapid Review Pinworms Transmission via ingestion of Enterobius vermicularis eggs Nocturnal pruritus ani Dx: tape test

Question: Is a clot in the superficial femoral vein a DVT?

Yes. The superficial femoral vein is a deep vein.


Ensembles d'études connexes

Nursing Process (PREPU Questions) CHP. 16 - PLANNING

View Set

BIOS 101, BIODIVERSITY AND EVOLUTION

View Set

Exam 3 Respiratory,Digestive, Nutrition,Metabolism, and Energy Balance

View Set

Ch. 3 Wong's - Evolve NCLEX Practice Q's

View Set

Cognitive approach to understanding

View Set

Financial Management in Sport Final

View Set

chapter 17: activity based costing

View Set